LSAT 12 – Section 1 – Question 05

You need a full course to see this video. Enroll now and get started in less than a minute.

Target time: 1:13

This is question data from the 7Sage LSAT Scorer. You can score your LSATs, track your results, and analyze your performance with pretty charts and vital statistics - all with a Free Account ← sign up in less than 10 seconds

Question
QuickView
Type Tags Answer
Choices
Curve Question
Difficulty
Psg/Game/S
Difficulty
Explanation
PT12 S1 Q05
+LR
Sufficient assumption +SA
A
19%
160
B
1%
165
C
1%
162
D
78%
165
E
1%
160
134
147
160
+Medium 147.398 +SubsectionMedium
This page shows a recording of a live class. We're working hard to create our standard, concise explanation videos for the questions in this PrepTest. Thank you for your patience!

This is a sufficient assumption question because of the question stem: “Which one of the following, if established, would help justify…” Note that there are two speakers; we’re specifically focused on validating Oscar’s argument.

Sufficient assumption questions tend to be very formal. We’re looking for a rule that would 100% validate the conclusion, specifically by bridging the premise and conclusion through the rule. Not only are we extrapolating the rule from our argument, but we’re also using that rule to render the argument “valid.” The way to prephrase our answer choice is by tying our premises and conclusion together into a rule: “If [premise] → then [conclusion].”

Oscar’s argument is given first. He’s being accused of plagiarizing the work of Myers but says it is unwarranted. What reason does he give for this? Why should we believe him? He first gives a concession: he used Myer’s work without attribution. Even though this is the case, he didn’t plagiarize because Myer gave him permission for this. So, the conclusion of Oscar’s argument is that the accusation that he’s guilty of plagiarism is unwarranted, and his premise is that he got permission in private correspondence to do use Myer’s work without attribution.

Millie’s argument is basically trying to weaken Oscar’s argument. We know this stimulus is for two different questions, so it’s like that question 6 has something to do with both of the speakers. On the older LSAT, it was very common for the test to give one stimulus for two different questions; this doesn’t happen anymore. We can basically ignore Millie’s claim because we’re trying to validate Oscar’s position. If you do end up reading it, the position Millie takes uncovers a useful implicit assumption that Oscar is making.

Back to the Oscar’s argument: does getting permission from the author to use their work without attribution exclude you from plagiarism? Do we know this from the information in our stimulus? No! This is the implicit assumption that Oscar is making. (This also happens to be the implicit assumption that Millie targets to weaken Oscar’s position). In order for this conclusion to be true, our rule would need to look something like “If I get permission in personal/private correspondence from the author to use their work without attribute, then I’m not guilty of plagiarizing.”

Answer Choice (A) This is one of the ways in which LSAT will routinely trick you – switching the sufficient and necessary conditions. Another way the LSAT will trick you is by putting the sufficient condition later in the sentence and putting the necessary condition first. Notice the “If” later on in the sentence; everything after that is our sufficient condition. With our rule, the direction of the conditional matters. This answer choice has “no right to quote” in the necessary condition, but we want “right to quote” or “not plagiarism.” We can also look at the sufficient condition: we’re looking for getting permission and our answer choice says, “the author hasn’t grant author any right.” This is out.

Answer Choice (B) This is another conditional in which the LSAT has put the sufficient condition later on in the sentence. Rearranged, it reads: “If the quote is more than a few sentences long, the writer of an article must attribute the text.” We can’t even satisfy the sufficient condition to trigger the conditional because we do not know how much Oscar pulled from Myer’s book. In addition, even if we could assume it’s just a few sentences, this would actually weaken the argument. This is also out.

Answer Choice (C) This isn’t correct: we start out we a blanket statement: “Plagiarism is never justified.” But then the answer gives us an exception: “but writers are justified in occasionally quoting without attribution…” Stop here. What would it need to be for this to be the correct answer choice? “When the author gives permission to use their writing without attribution.” That would be great! What does our answer choice say? “If the work has not been published.” This does not apply to our stimulus because we don’t know if it hasn’t/has been published.

Correct Answer Choice (D) The necessary condition is presented first, so if we rearrange the order, we get: “if a writer relinquishes their exclusive right to their work, then another author is entitled to quote freely without attribution.” The “relinquished their exclusive right” is a little dramatic and fancy but saying in private correspondence that someone else can use their work without attribution is relinquishing an exclusive right.

Answer Choice (E) “Quote without attribution what they themselves have written” is the problem here. We’re not talking about what Oscar quotes from his own work; he’s quoting another author. This is out.

Take PrepTest

Review Results

Leave a Reply